Number Theory 21

(IMO 1997)
[Bài toán]: Tìm nghiệm nguyên dương của phương trình: a^{b^2}=b^a
                                                              Lời giải 
Đặt \gcd(a,b)=d\Rightarrow a=dm,b=dn(\gcd(m,n)=1)
\Rightarrow [(dm)^{dn^2}]^d=[(dn)^m]^d\Rightarrow (dm)^{dn^2}=(dn)^m\Rightarrow d^{dn^2}.m^{dn^2}=d^m.n^m(1)
\blacktriangleright TH1: dn^2=m\Rightarrow m^{dn^2}=n^m\gcd(m,n)=1\Rightarrow m=n=1dn^2=m\Rightarrow d=1\Rightarrow (a,b)=(1;1)
\blacktriangleright TH2: dn^2>m\Rightarrow d^{dn^2-m}.m^{dn^2}=n^m
Thấy m^{dn^2}\mid n^m\gcd(m,n)=1\Rightarrow m=1
\Rightarrow d^{dn^2-1}=n^1=n
• Xét d=1\Rightarrow n=1 \Rightarrow (a,b)=(1;1)
• Xét d\geq 2\Rightarrow n\geq 2. Dùng quy nạp, ta sẽ CM: 2^{2n^2-1}>n\qquad (2)
Giả sử (2) \text{True}\forall n=k, ta sẽ CM \forall n=k+1,(3) vẫn đúng.
Thật vậy, ta có: 2^{2k^2-1}>k\Rightarrow 2^{2k^2-1}\geq k+1
Ta có: 2^{2(k+1)^2-1}=2^{2k^2+4k+1}=2^{2k^2-1}.2^{4k+2}\geq (k+1).2^{4k+2}\geq k+1
Do đó (2) được CM \Rightarrow Trường hợp này không xảy ra.
\blacktriangleright TH3: Xét dn^2<m\Rightarrow (2)\Leftrightarrow m^{dn^2}=d^{m-dn^2}.n^m \Rightarrow n^m\mid m^{dn^2}\gcd(m,n)=1\Rightarrow n=1
\Rightarrow (2)\Leftrightarrow m^d=d^{m-d}. Theo bài Number Theory 8
\Rightarrow (m;d)\in \{(1;1);(9;3);(8;2)\}\Rightarrow (a,b)\in \{(1;1);(27;3);(16;2)\}
Kết luận: Nghiệm của phương trình là (a,b)\in \{(1;1);(27;3);(16;2)\} \blacksquare

Leave a comment